• Anúncio Global
    Respostas
    Exibições
    Última mensagem

[Limite] Gostaria de saber se posso operar dessa maneira.

[Limite] Gostaria de saber se posso operar dessa maneira.

Mensagempor ravi » Ter Out 09, 2012 10:50

Olá amigos, resolvi esse limite usando todas as propriedades da matemática corretamente, mas não sei se na teoria de limites existe algo que me impeça de faze-lo dessa maneira:
obs: Me parece que o editor de fórmulas está com problemas, ou eu não soube utiliza-lo corretamente.



\lim_{+\propto}(x-\sqrt[]{x+3})


=\frac{(x-\sqrt[]{x+3})(x+\sqrt[]{x+3})}{x+\sqrt[]{x+3}} Devido a indeterminação que ocorre acima, multipliquei pelo conjugado.


=\frac{{x}^{2}(1-\frac{1}{x}-\frac{3}{x}}{x+\sqrt[]{x(1+\frac{3}{x})}} Daí então cheguei a um resultado e já simplifiquei dessa maneira.


=\frac{\sqrt[1]{{x}^{2}}}{\sqrt[]{x}+x} Daí eu resolvi fazer essa divisão de radicais porque em cima não consegui evitar a indeterminação. (Essa é minha dúvida não sei se posso fazer isso).


=\sqrt[3]{x}+x = +\propto Chegando ao resultado da divisão de radicais e finalmente ao resultado final(+infinito).

Então essa é minha dúvida amigos, não sei se existe algo na teoria do assunto de limites que me impeça de fazer aquela divisão de radicais(na segunda sentença de baixo para cima), mas eu fiz a divisão utilizando as propriedades e evitei a indeterminação.

Muito obrigado, e para o pessoal do nordeste fiquem atentos que em novembro tem três congressos, um em Salvador, um em Natal e outro em João Pessoa, entre no site da SBM e confira.

Um abraço a todos.
ravi
Novo Usuário
Novo Usuário
 
Mensagens: 9
Registrado em: Ter Jan 31, 2012 13:20
Formação Escolar: ENSINO MÉDIO
Área/Curso: licenciatura em matemática
Andamento: cursando

Re: [Limite] Gostaria de saber se posso operar dessa maneira

Mensagempor LuizAquino » Ter Out 09, 2012 11:40

ravi escreveu:Olá amigos, resolvi esse limite usando todas as propriedades da matemática corretamente, mas não sei se na teoria de limites existe algo que me impeça de faze-lo dessa maneira:
obs: Me parece que o editor de fórmulas está com problemas, ou eu não soube utiliza-lo corretamente.



\lim_{+\propto}(x-\sqrt[]{x+3})


=\frac{(x-\sqrt[]{x+3})(x+\sqrt[]{x+3})}{x+\sqrt[]{x+3}} Devido a indeterminação que ocorre acima, multipliquei pelo conjugado.


=\frac{{x}^{2}(1-\frac{1}{x}-\frac{3}{x}}{x+\sqrt[]{x(1+\frac{3}{x})}} Daí então cheguei a um resultado e já simplifiquei dessa maneira.


=\frac{\sqrt[1]{{x}^{2}}}{\sqrt[]{x}+x} Daí eu resolvi fazer essa divisão de radicais porque em cima não consegui evitar a indeterminação. (Essa é minha dúvida não sei se posso fazer isso).


=\sqrt[3]{x}+x = +\propto Chegando ao resultado da divisão de radicais e finalmente ao resultado final(+infinito).

Então essa é minha dúvida amigos, não sei se existe algo na teoria do assunto de limites que me impeça de fazer aquela divisão de radicais(na segunda sentença de baixo para cima), mas eu fiz a divisão utilizando as propriedades e evitei a indeterminação.


Na verdade, você não usou "todas as propriedades da matemática corretamente".

Em primeiro lugar, você deveria escrever as notações de forma adequada. Note que na sua resolução você não escreveu a notação \lim_{x\to+\infty} em todos os passos. Você deve escrever essa notação em todos os passos, exceto no último, quando você efetivamente calcula o valor do limite. Desse modo, você deveria ter escrito:

\lim_{x\to+\infty} x - \sqrt{x + 3}= \lim_{x\to+\infty} \frac{\left(x - \sqrt{x + 3}\right)\left(x + \sqrt{x + 3}\right)}{\left(x + \sqrt{x + 3}\right)}

= \lim_{x\to+\infty} \frac{x^2\left(1 - \frac{1}{x} - \frac{3}{x^2}\right)}{x + \sqrt{x\left(1 + \frac{3}{x}\right)}}

Aqui note também que você escreveu -3/x no lugar de -3/(x²).

E em segundo lugar, depois desse passo você efetuou uma "divisão de radicais" que não existe.

O correto na verdade seria fazer:

= \lim_{x\to+\infty} \frac{x^2\left(1 - \frac{1}{x} - \frac{3}{x^2}\right)}{x\left(1 + \sqrt{\frac{1}{x} + \frac{3}{x^2}}\right)}

= \lim_{x\to+\infty} \frac{x\left(1 - \frac{1}{x} - \frac{3}{x^2}\right)}{1 + \sqrt{\frac{1}{x} + \frac{3}{x^2}}}

=  \frac{(+\infty)\left(1 - 0 - 0\right)}{1 + \sqrt{0 + 0}} = +\infty
professoraquino.com.br | youtube.com/LCMAquino | @lcmaquino

"Sem esforço, não há ganho."
Dito popular.
Avatar do usuário
LuizAquino
Colaborador Moderador - Professor
Colaborador Moderador - Professor
 
Mensagens: 2654
Registrado em: Sex Jan 21, 2011 09:11
Localização: Teófilo Otoni - MG
Formação Escolar: PÓS-GRADUAÇÃO
Área/Curso: Mestrado - Modelagem Computacional
Andamento: formado

Re: [Limite] Gostaria de saber se posso operar dessa maneira

Mensagempor ravi » Ter Out 09, 2012 12:31

Obrigado amigo, porém não entendi como você chegou a:


x(1+\sqrt[]{\frac{1}{x}+\frac{3}{{x}^{2}}})


e também não entendi porque a divisão que eu fiz não existe, se:

\frac{{x}^{2}}{\sqrt[]{x}}=\frac{\sqrt[1]{{x}^{2}}}{\sqrt[]{x}}=\frac{\sqrt[]{{x}^{4}}}{\sqrt[]{x}}=\sqrt[]{\frac{{x}^{4}}{x}}=\sqrt[]{{x}^{3}} Tem algum erro nesta sentença?

Muito obrigado!
ravi
Novo Usuário
Novo Usuário
 
Mensagens: 9
Registrado em: Ter Jan 31, 2012 13:20
Formação Escolar: ENSINO MÉDIO
Área/Curso: licenciatura em matemática
Andamento: cursando

Re: [Limite] Gostaria de saber se posso operar dessa maneira

Mensagempor LuizAquino » Ter Out 09, 2012 14:14

ravi escreveu:Obrigado amigo, porém não entendi como você chegou a:

x(1+\sqrt[]{\frac{1}{x}+\frac{3}{{x}^{2}}})


Note que:

x + \sqrt{x\left(1 + \frac{3}{x}\right)} = x\left[1  + \frac{1}{x}\sqrt{x\left(1 + \frac{3}{x}\right)}\right]

= x\left[1  + \sqrt{\frac{x}{x^2}\left(1 + \frac{3}{x}\right)}\right]

= x\left(1  + \sqrt{\frac{1}{x} + \frac{3}{x^2}}\right)

ravi escreveu:e também não entendi porque a divisão que eu fiz não existe, se:

\frac{{x}^{2}}{\sqrt[]{x}}=\frac{\sqrt[1]{{x}^{2}}}{\sqrt[]{x}}=\frac{\sqrt[]{{x}^{4}}}{\sqrt[]{x}}=\sqrt[]{\frac{{x}^{4}}{x}}=\sqrt[]{{x}^{3}} Tem algum erro nesta sentença?


Nessa sentença não há. Mas vejamos o que você fez:

ravi escreveu:=\frac{\sqrt[1]{{x}^{2}}}{\sqrt[]{x}+x} Daí eu resolvi fazer essa divisão de radicais porque em cima não consegui evitar a indeterminação. (Essa é minha dúvida não sei se posso fazer isso).


=\sqrt[3]{x}+x = +\propto Chegando ao resultado da divisão de radicais e finalmente ao resultado final(+infinito).


Com isso você está dizendo que \frac{\sqrt[1]{x^2}}{\sqrt{x}+x} seria o mesmo que \frac{\sqrt[1]{x^2}}{\sqrt{x}} + \frac{\sqrt[1]{x^2}}{x} , mas isso está errado. E mesmo ao cometer esse erro, você ainda cometeu outro: escreveu \sqrt[3]{x} ao invés de \sqrt{x^3} .
professoraquino.com.br | youtube.com/LCMAquino | @lcmaquino

"Sem esforço, não há ganho."
Dito popular.
Avatar do usuário
LuizAquino
Colaborador Moderador - Professor
Colaborador Moderador - Professor
 
Mensagens: 2654
Registrado em: Sex Jan 21, 2011 09:11
Localização: Teófilo Otoni - MG
Formação Escolar: PÓS-GRADUAÇÃO
Área/Curso: Mestrado - Modelagem Computacional
Andamento: formado

Re: [Limite] Gostaria de saber se posso operar dessa maneira

Mensagempor ravi » Qui Out 11, 2012 13:28

Obrigado, entendi.

Mas, se ao invés de:

=\frac{{x}^{2}(1-\frac{1}{x}-\frac{3}{x})}{x+\sqrt[]{x(1+\frac{3}{x})}}


=\frac{\sqrt[1]{{x}^{2}}}{\sqrt[]{x}+x}=\sqrt[]{{x}^{3}}+x = +\propto


Eu fizesse:

=\frac{{x}^{2}(1-\frac{1}{x}-\frac{3}{x})}{x+\sqrt[]{x(1+\frac{3}{x})}}


=\frac{x(1-\frac{1}{x}-\frac{3}{x})}{\sqrt[]{x(1+\frac{3}{x})}}


=\frac{x}{\sqrt[]{x}}

=\sqrt[]{x}=+\propto
ravi
Novo Usuário
Novo Usuário
 
Mensagens: 9
Registrado em: Ter Jan 31, 2012 13:20
Formação Escolar: ENSINO MÉDIO
Área/Curso: licenciatura em matemática
Andamento: cursando

Re: [Limite] Gostaria de saber se posso operar dessa maneira

Mensagempor LuizAquino » Qui Out 11, 2012 23:37

ravi escreveu:Obrigado, entendi.

Mas, se ao invés de:

=\frac{{x}^{2}(1-\frac{1}{x}-\frac{3}{x})}{x+\sqrt[]{x(1+\frac{3}{x})}}


=\frac{\sqrt[1]{{x}^{2}}}{\sqrt[]{x}+x}=\sqrt[]{{x}^{3}}+x = +\propto


Eu fizesse:

=\frac{{x}^{2}(1-\frac{1}{x}-\frac{3}{x})}{x+\sqrt[]{x(1+\frac{3}{x})}}


=\frac{x(1-\frac{1}{x}-\frac{3}{x})}{\sqrt[]{x(1+\frac{3}{x})}}


=\frac{x}{\sqrt[]{x}}

=\sqrt[]{x}=+\propto


Continuaria errado.

Além de cometer os mesmos erros anteriores (não escrever a notação de limites e colocar -3/x onde deveria ser -3/(x²)), você ainda cometeu o erro de efetuar uma simplificação que não existe. Por exemplo, note que a expressão numérica \frac{5^2\cdot 3}{5 + 2} não é igual a \frac{5\cdot 3}{2} .
professoraquino.com.br | youtube.com/LCMAquino | @lcmaquino

"Sem esforço, não há ganho."
Dito popular.
Avatar do usuário
LuizAquino
Colaborador Moderador - Professor
Colaborador Moderador - Professor
 
Mensagens: 2654
Registrado em: Sex Jan 21, 2011 09:11
Localização: Teófilo Otoni - MG
Formação Escolar: PÓS-GRADUAÇÃO
Área/Curso: Mestrado - Modelagem Computacional
Andamento: formado


Voltar para Cálculo: Limites, Derivadas e Integrais

 



  • Tópicos relacionados
    Respostas
    Exibições
    Última mensagem

Quem está online

Usuários navegando neste fórum: Nenhum usuário registrado e 8 visitantes

 



Assunto: [Função] do primeiro grau e quadratica
Autor: Thassya - Sáb Out 01, 2011 16:20

1) Para que os pontos (1,3) e (-3,1) pertençam ao grafico da função f(X)=ax + b ,o valor de b-a deve ser ?

2)Qual o maior valor assumido pela função f : [-7 ,10] em R definida por f(x) = x ao quadrado - 5x + 9?

3) A função f, do primeiro grau, é definida pos f(x)= 3x + k para que o gráfico de f corte o eixo das ordenadas no ponto de ordenada 5 é?


Assunto: [Função] do primeiro grau e quadratica
Autor: Neperiano - Sáb Out 01, 2011 19:46

Ola

Qual as suas dúvidas?

O que você não está conseguindo fazer?

Nos mostre para podermos ajudar

Atenciosamente


Assunto: [Função] do primeiro grau e quadratica
Autor: joaofonseca - Sáb Out 01, 2011 20:15

1)Dados dois pontos A=(1,3) e B=(-3,1) de uma reta, é possivel definir a sua equação.

y_{b}-y_{a}=m(x_{b}-x_{a})

1-3=m(-3-1) \Leftrightarrow -2=-4m \Leftrightarrow m=\frac{2}{4} \Leftrightarrow m=\frac{1}{2}

Em y=mx+b substitui-se m, substitui-se y e x por um dos pares ordenados, e resolve-se em ordem a b.

3=\frac{1}{2} \cdot 1+b\Leftrightarrow 3-\frac{1}{2}=b \Leftrightarrow b=\frac{5}{2}



2)Na equação y=x^2-5x+9 não existem zeros.Senão vejamos

Completando o quadrado,

(x^2-5x+\frac{25}{4})+9-\frac{25}{4} =0\Leftrightarrow (x-\frac{5}{2})^2+\frac{11}{4}=0

As coordenadas do vertice da parabola são (\frac{5}{2},\frac{11}{4})

O eixo de simetria é a reta x=\frac{5}{2}.Como se pode observar o vertice está acima do eixo Ox, estando parabola virada para cima, o vertice é um mínimo absoluto.Então basta calcular a função para os valores dos extremos do intervalo.

f(-7)=93
f(10)=59